Как приготовить раствор для: Цементный раствор — пропорции: как развести и сколько песка и цемента в 1 м3, соотношение частей и расход

Содержание

Цементный раствор — пропорции: как развести и сколько песка и цемента в 1 м3, соотношение частей и расход

Цемент – это основной строительный материал, который используется практически во всех отраслях народного хозяйства. С помощью данного вещества можно получить очень прочные продукты, способные выдерживать высокие нагрузки и противостоять внешним воздействиям. Но все эти характеристики зависят также и от используемых компонентов, и технологии приготовления. Цементные растворы широко применяются в строительстве, так как позволяют упростить много операций.

Особенности

Цементные растворы представляют собой искусственные смеси, которые после застывания образуют прочную структуру.

Состоит подобный продукт из нескольких основных компонентов.

  • Песок. Он используется в качестве основного компонента, так как объединяет в себе мелкую структуру и относительно высокую прочность. Для приготовления растворов могут использовать речной или карьерный песок. Первый тип материала применяется при монолитном строительстве, позволяя получить очень прочные продукты.
  • Вода. Данный компонент нужен для связывания песка и цемента. Количество жидкости подбирается в зависимости от марки и предназначения раствора.
  • Цемент. Это основное вещество, которое отличается высокой адгезией с другими материалами. Сегодня существует несколько марок цемента, предназначенного для эксплуатации в различных условиях. Отличаются они показателями прочности.
  • Пластификаторы. Технически это различные виды примесей, которые предназначаются для изменения физических или химических свойств раствора. Они используются не так часто, так как это может значительно увеличивать стоимость продукта.

Подобную продукцию используют для решения следующих видов задач:

  • оштукатуривание – некоторыми растворами покрывают стены для защиты строительного материала, а также с целью выравнивания основания;
  • кладка – цементные смеси прекрасно связывают между собой кирпич или газоблок, поэтому их используют в качестве своеобразного клея, располагающегося внутри каждого шва;
  • создание железобетонных конструкций.

Виды составов и требования

Основной характеристикой цементного раствора является его прочность. Она обусловлена соотношением цемента и песка. Состав продукта может изменяться штучно, что позволяет получить несколько видов смесей. Каждый из них предназначается для использования в определенных условиях. Поэтому важно правильно готовить продукцию при строительстве различных объектов.

Типы

Одним из критериев разделения цементных смесей на виды являются пропорции внутренних компонентов. Стоит обратить внимание, что в одном составе может присутствовать только одна марка цемента. Но они могут также изменяться, так как прочность будет зависеть уже только от концентрации компонентов. Условно их разделяют на несколько марок.

  • М100 (М150) – эти смеси отличаются незначительной прочностью. Для их приготовления можно использовать цемент марок М200–М500. Но при этом необходимо правильно подбирать пропорции цементно-песчаных компонентов.
  • М200 – это один из самых распространенных видов растворов. Его используют очень часто в быту для строительства дорожек и или формирования покрытий, которые не поддаются значительным нагрузкам. Сохнет данная смесь относительно быстро, но при этом требует соблюдения определенных микроклиматических условий.
  • М300 – данный вид раствора можно уже отнести к бетонным типам. Он используется для приготовления бетонов, из которых затем изготавливают прочные плиты перекрытия, заливают фундаменты и много другого.
  • М400
    – это прочный бетон, который состоит из качественных марок цемента (М350, М400, М500). Используют его в строительстве фундаментов для многоэтажных домов. Данный раствор составляет основу для изготовления железобетонных плит перекрытия и других подобных изделий.
  • М500 – это самый прочный бетон, который способен выдерживать очень высокие нагрузки. Он сохраняет свои первоначальные свойства на протяжении многих лет и при воздействии различных раздражителей.

Добавки

Качество цементного раствора зависит практически от всех его составляющих, которые присутствуют внутри. Иногда свойств песчано-цементной смеси недостаточно, поэтому нужно адаптировать их под определенные условия.

Решается эта проблема с помощью добавления в состав различных примесей. С помощью подобных присадок получают так называемое жидкое стекло. Эти продукты используются для оштукатуривания стен и других поверхностей.

Сегодня в качестве добавок для цементных растворов используют несколько продуктов.

  • Известь. В качестве добавок применяют только гашенные ее виды. Внедрение этого вещества позволяет немного увеличить паропроницаемость и прочность. Но чтобы приготовить подобную продукцию, следует соблюдать точные пропорции. Очень часто на основе извести изготавливают штукатурки, которые прекрасно наносятся на стены.
  • ПВА. Клей улучшает адгезию и пластичность раствора. Важно правильно подбирать концентрацию добавки, чтобы получить хорошую смесь.
  • Моющие средства. Подобные продукты влияют на пластичность раствора. Добавляют их в состав только после воды. Здесь также обязательно соблюдается точная доза примеси на единицу объема.
  • Сажа или графит. Эти вещества практически не влияют на физические свойства смеси. Используются они только в качестве красителей для изменения цвета готового продукта.

Соотношение песка и цемента

Приготовить цементно-песчаный раствор можно даже в домашних условиях, так как состоит он из доступных компонентов. Приобрести их довольно легко практически в любом строительном магазине. Но отличаются растворы соотношением цемента и песка, от которых и зависит расход и физические характеристики материала.

Кирпичная кладка

Скрепление кирпичей – это одна из основных задач цементных растворов. Для таких целей используют не особо прочные марки (до М400). Для получения подобной смеси специалисты рекомендуют использовать песок средней фракции с минимальным уровнем влажности. Приготовить кладочный раствор можно используя различные марки цемента. Но при этом будет уже изменяться соотношение цемента и песка. Некоторые пропорции представлены в таблице 1.

Таблица 1. Соотношения компонентов в зависимости от марки цемента

Обратите внимание, что расчет желательно проводить согласно только одних единиц измерения. В большинстве случаев все части рассчитывают на 1 м³. Но при этом массам различных материалов в кубе может отличаться.

Приготовление бетона

Бетонные конструкции также очень часто используются в современной промышленности. Эти материалы изготавливаются на заводах или непосредственно на строительных площадках. Прочность таких изделий также зависит от цемента, который планируется использовать. Технически бетон можно изготовить и из раствора марки М100, но он не будет выдерживать нагрузки, и отличаться минимальным сроком службы.

Еще одной особенностью бетонов является наличие в составе щебня и других вспомогательных компонентов. Они внедряются с целью изменения технических характеристик продукта.

Следует отметить, что смешиваться они могут в различных комбинациях, что зависит от среды использования бетона.

Сегодня многие специалисты используют такое соотношение компонентов бетонных растворов, как:

  • 4 части щебня;
  • 1 часть цемента;
  • 2 части песка;
  • ½ части воды.

Обратите внимание, что пропорции могут изменяться, если еще планируется использовать различные полимерные добавки. В таких случаях желательно обращать внимание на рекомендации производителей данных примесей.

Для штукатурки и стяжки

Заливка пола очень часто предполагает использование относительно жидких цементных растворов. Такая консистенция позволяет равномерно распределить смесь на основании и получить горизонтальную поверхность. Штукатурка же практически всегда состоит только из чистого песка, цемента и воды. Ее густота может быть разной, так как все зависит от того, где ее планируется использовать.

Самой распространенной пропорцией для получения штукатурных смесей является отношение цемента к песку 1: 5. Консистенция адаптируется под потребности мастера.

Особое внимание следует уделить стяжкам, которые поддаются значительным и постоянным нагрузкам. Для таких поверхностей следует использовать материалы, у которых пороговая прочность не меньше 10 МПа. Достигается это за счет использования бетонов марки не ниже М150. Пропорция приготовления раствора для стяжки зависит от следующих факторов:

  • использование смесей с целью сокрытия различных коммуникационных элементов;
  • толщина выравнивания поверхности. Если нужно просто укрепить пол с небольшими перепадами, тогда применяют более жидкие составы. Для более толстых слоев желательно использовать прочные виды растворов.

Таблица 2. Пропорции песка и цемента в стяжках

Обратите внимание, что пропорции компонентов в большинстве случаев повторяются. Но при этом прочность получаемого раствора на выходе отличается. Это важно учитывать, если продукция будет применяться в специфических условиях эксплуатации.

Как правильно развести?

Процесс приготовления цементных растворов предполагает смешивание всех компонентов в определенной последовательности. Описать подобную процедуру можно несколькими последовательными шагами.

  • В первую очередь нужно определиться с типом раствора, который нужен. При этом обращают внимание на прочность результирующей смеси. Если важен этот показатель, следует провести дополнительный расчет всех составляющих. Особое внимание следует обращать на нормы или стандарты.
  • На данном этапе смешивают сухие компоненты, объем которых измеряется в 1 м³ или других подобных единицах. Перед тем как замесить раствор, следует тщательно перемешать песок и цемент, чтобы получить равномерную смесь. Поэтому так важно использовать сухие вещества.
  • Когда подготовка прошла успешно, можно разводить смесь. Для этого постепенно в нее добавляют воду и тщательно смешивают все компоненты. Лучше всего использовать бетономешалки или другие механические приспособления. Консистенция раствора регулируется с помощью жидкости.

Советы и рекомендации

Приготовление цементного раствора является простой операцией. При ее осуществлении все-таки рекомендовано соблюдать несколько простых правил, рекомендуемых производителем и опытными строителями, такие как:

  • если смесь должна быть пластичной, для этого нужно добавлять в нее жидкое мыло. Его смешивать нужно предварительно с водой;
  • следует добавлять воду небольшими порциями. Таким образом, можно контролировать густоту смеси, которая очень важна для стяжек или кладки;
  • при строительстве обязательно нужно учитывать марку самого кирпича или другого материала. Специалисты рекомендуют готовить такие смеси, которые по данным параметрам должны совпадать. Это позволит получить однородную структуру стены, которая будет отличаться прочностью;
  • для повышения теплоизоляционных характеристик штукатурок стоит добавлять в их состав перлит. При этом им нужно заменять определенную часть песка;
  • рекомендуется использовать только свежий цемент, структура которого не содержит комков. Это гарантирует высокую адгезию и равномерное смешивание.

Цементный раствор – это прекрасный материал, позволяющий получить прочные конструкции. Правильно приготовленная смесь – это залог долговечности практически любого строения и его основания.

Подробнее о пропорциях цементного раствора вы узнаете из следующего видео.

Цементный раствор — пропорции: как развести и сколько песка и цемента в 1 м3, соотношение частей и расход

Цемент – это основной строительный материал, который используется практически во всех отраслях народного хозяйства. С помощью данного вещества можно получить очень прочные продукты, способные выдерживать высокие нагрузки и противостоять внешним воздействиям. Но все эти характеристики зависят также и от используемых компонентов, и технологии приготовления. Цементные растворы широко применяются в строительстве, так как позволяют упростить много операций.

Особенности

Цементные растворы представляют собой искусственные смеси, которые после застывания образуют прочную структуру. Состоит подобный продукт из нескольких основных компонентов.

  • Песок. Он используется в качестве основного компонента, так как объединяет в себе мелкую структуру и относительно высокую прочность. Для приготовления растворов могут использовать речной или карьерный песок. Первый тип материала применяется при монолитном строительстве, позволяя получить очень прочные продукты.
  • Вода. Данный компонент нужен для связывания песка и цемента. Количество жидкости подбирается в зависимости от марки и предназначения раствора.
  • Цемент. Это основное вещество, которое отличается высокой адгезией с другими материалами. Сегодня существует несколько марок цемента, предназначенного для эксплуатации в различных условиях. Отличаются они показателями прочности.
  • Пластификаторы. Технически это различные виды примесей, которые предназначаются для изменения физических или химических свойств раствора. Они используются не так часто, так как это может значительно увеличивать стоимость продукта.

Подобную продукцию используют для решения следующих видов задач:

  • оштукатуривание – некоторыми растворами покрывают стены для защиты строительного материала, а также с целью выравнивания основания;
  • кладка – цементные смеси прекрасно связывают между собой кирпич или газоблок, поэтому их используют в качестве своеобразного клея, располагающегося внутри каждого шва;
  • создание железобетонных конструкций.

Виды составов и требования

Основной характеристикой цементного раствора является его прочность. Она обусловлена соотношением цемента и песка. Состав продукта может изменяться штучно, что позволяет получить несколько видов смесей. Каждый из них предназначается для использования в определенных условиях. Поэтому важно правильно готовить продукцию при строительстве различных объектов.

Типы

Одним из критериев разделения цементных смесей на виды являются пропорции внутренних компонентов. Стоит обратить внимание, что в одном составе может присутствовать только одна марка цемента. Но они могут также изменяться, так как прочность будет зависеть уже только от концентрации компонентов. Условно их разделяют на несколько марок.

  • М100 (М150) – эти смеси отличаются незначительной прочностью. Для их приготовления можно использовать цемент марок М200–М500. Но при этом необходимо правильно подбирать пропорции цементно-песчаных компонентов.
  • М200 – это один из самых распространенных видов растворов. Его используют очень часто в быту для строительства дорожек и или формирования покрытий, которые не поддаются значительным нагрузкам. Сохнет данная смесь относительно быстро, но при этом требует соблюдения определенных микроклиматических условий.
  • М300 – данный вид раствора можно уже отнести к бетонным типам. Он используется для приготовления бетонов, из которых затем изготавливают прочные плиты перекрытия, заливают фундаменты и много другого.
  • М400 – это прочный бетон, который состоит из качественных марок цемента (М350, М400, М500). Используют его в строительстве фундаментов для многоэтажных домов. Данный раствор составляет основу для изготовления железобетонных плит перекрытия и других подобных изделий.
  • М500 – это самый прочный бетон, который способен выдерживать очень высокие нагрузки. Он сохраняет свои первоначальные свойства на протяжении многих лет и при воздействии различных раздражителей.

Добавки

Качество цементного раствора зависит практически от всех его составляющих, которые присутствуют внутри. Иногда свойств песчано-цементной смеси недостаточно, поэтому нужно адаптировать их под определенные условия.

Решается эта проблема с помощью добавления в состав различных примесей. С помощью подобных присадок получают так называемое жидкое стекло. Эти продукты используются для оштукатуривания стен и других поверхностей.

Сегодня в качестве добавок для цементных растворов используют несколько продуктов.

  • Известь. В качестве добавок применяют только гашенные ее виды. Внедрение этого вещества позволяет немного увеличить паропроницаемость и прочность. Но чтобы приготовить подобную продукцию, следует соблюдать точные пропорции. Очень часто на основе извести изготавливают штукатурки, которые прекрасно наносятся на стены.
  • ПВА. Клей улучшает адгезию и пластичность раствора. Важно правильно подбирать концентрацию добавки, чтобы получить хорошую смесь.
  • Моющие средства. Подобные продукты влияют на пластичность раствора. Добавляют их в состав только после воды. Здесь также обязательно соблюдается точная доза примеси на единицу объема.
  • Сажа или графит. Эти вещества практически не влияют на физические свойства смеси. Используются они только в качестве красителей для изменения цвета готового продукта.

Соотношение песка и цемента

Приготовить цементно-песчаный раствор можно даже в домашних условиях, так как состоит он из доступных компонентов. Приобрести их довольно легко практически в любом строительном магазине. Но отличаются растворы соотношением цемента и песка, от которых и зависит расход и физические характеристики материала.

Кирпичная кладка

Скрепление кирпичей – это одна из основных задач цементных растворов. Для таких целей используют не особо прочные марки (до М400). Для получения подобной смеси специалисты рекомендуют использовать песок средней фракции с минимальным уровнем влажности. Приготовить кладочный раствор можно используя различные марки цемента. Но при этом будет уже изменяться соотношение цемента и песка. Некоторые пропорции представлены в таблице 1.

Таблица 1. Соотношения компонентов в зависимости от марки цемента

Обратите внимание, что расчет желательно проводить согласно только одних единиц измерения. В большинстве случаев все части рассчитывают на 1 м³. Но при этом массам различных материалов в кубе может отличаться.

Приготовление бетона

Бетонные конструкции также очень часто используются в современной промышленности. Эти материалы изготавливаются на заводах или непосредственно на строительных площадках. Прочность таких изделий также зависит от цемента, который планируется использовать. Технически бетон можно изготовить и из раствора марки М100, но он не будет выдерживать нагрузки, и отличаться минимальным сроком службы.

Еще одной особенностью бетонов является наличие в составе щебня и других вспомогательных компонентов. Они внедряются с целью изменения технических характеристик продукта.

Следует отметить, что смешиваться они могут в различных комбинациях, что зависит от среды использования бетона.

Сегодня многие специалисты используют такое соотношение компонентов бетонных растворов, как:

  • 4 части щебня;
  • 1 часть цемента;
  • 2 части песка;
  • ½ части воды.

Обратите внимание, что пропорции могут изменяться, если еще планируется использовать различные полимерные добавки. В таких случаях желательно обращать внимание на рекомендации производителей данных примесей.

Для штукатурки и стяжки

Заливка пола очень часто предполагает использование относительно жидких цементных растворов. Такая консистенция позволяет равномерно распределить смесь на основании и получить горизонтальную поверхность. Штукатурка же практически всегда состоит только из чистого песка, цемента и воды. Ее густота может быть разной, так как все зависит от того, где ее планируется использовать.

Самой распространенной пропорцией для получения штукатурных смесей является отношение цемента к песку 1: 5. Консистенция адаптируется под потребности мастера.

Особое внимание следует уделить стяжкам, которые поддаются значительным и постоянным нагрузкам. Для таких поверхностей следует использовать материалы, у которых пороговая прочность не меньше 10 МПа. Достигается это за счет использования бетонов марки не ниже М150. Пропорция приготовления раствора для стяжки зависит от следующих факторов:

  • использование смесей с целью сокрытия различных коммуникационных элементов;
  • толщина выравнивания поверхности. Если нужно просто укрепить пол с небольшими перепадами, тогда применяют более жидкие составы. Для более толстых слоев желательно использовать прочные виды растворов.

Таблица 2. Пропорции песка и цемента в стяжках

Обратите внимание, что пропорции компонентов в большинстве случаев повторяются. Но при этом прочность получаемого раствора на выходе отличается. Это важно учитывать, если продукция будет применяться в специфических условиях эксплуатации.

Как правильно развести?

Процесс приготовления цементных растворов предполагает смешивание всех компонентов в определенной последовательности. Описать подобную процедуру можно несколькими последовательными шагами.

  • В первую очередь нужно определиться с типом раствора, который нужен. При этом обращают внимание на прочность результирующей смеси. Если важен этот показатель, следует провести дополнительный расчет всех составляющих. Особое внимание следует обращать на нормы или стандарты.
  • На данном этапе смешивают сухие компоненты, объем которых измеряется в 1 м³ или других подобных единицах. Перед тем как замесить раствор, следует тщательно перемешать песок и цемент, чтобы получить равномерную смесь. Поэтому так важно использовать сухие вещества.
  • Когда подготовка прошла успешно, можно разводить смесь. Для этого постепенно в нее добавляют воду и тщательно смешивают все компоненты. Лучше всего использовать бетономешалки или другие механические приспособления. Консистенция раствора регулируется с помощью жидкости.

Советы и рекомендации

Приготовление цементного раствора является простой операцией. При ее осуществлении все-таки рекомендовано соблюдать несколько простых правил, рекомендуемых производителем и опытными строителями, такие как:

  • если смесь должна быть пластичной, для этого нужно добавлять в нее жидкое мыло. Его смешивать нужно предварительно с водой;
  • следует добавлять воду небольшими порциями. Таким образом, можно контролировать густоту смеси, которая очень важна для стяжек или кладки;
  • при строительстве обязательно нужно учитывать марку самого кирпича или другого материала. Специалисты рекомендуют готовить такие смеси, которые по данным параметрам должны совпадать. Это позволит получить однородную структуру стены, которая будет отличаться прочностью;
  • для повышения теплоизоляционных характеристик штукатурок стоит добавлять в их состав перлит. При этом им нужно заменять определенную часть песка;
  • рекомендуется использовать только свежий цемент, структура которого не содержит комков. Это гарантирует высокую адгезию и равномерное смешивание.

Цементный раствор – это прекрасный материал, позволяющий получить прочные конструкции. Правильно приготовленная смесь – это залог долговечности практически любого строения и его основания.

Подробнее о пропорциях цементного раствора вы узнаете из следующего видео.

глиняный, состав, пропорции, как сделать, приготовить своими руками, каким штукатурить печные трубы, видео-инструкция, фото и цена

Вот и наступили времена, когда традиционные печи стали казаться дремучим пережитком прошлого, ведь, тотальная газификация частного жилья практически вытеснила альтернативное топливо. За долгие годы они практически не претерпели особых изменений в отделке, так как новые материалы фактически не разрабатывались.

На фото – проведение оштукатуривания печи

Это, конечно, напрасно, так как печь – достаточно надежный источник тепла, который не страдает от перебоев подачи газа, электроэнергии, дизтоплива или пеллет. Достаточно просто сделать своевременный запас дров и угля, так что услуги печников, скорее всего, снова будут в цене. В статье ниже мы попытаемся раскрыть один из секретов, каким должен быть раствор для оштукатуривания печей.

Этот вопрос неизбежно возникнет при сооружении нового или ремонте старого отопительного прибора. Согласитесь, условия у отделки будут сродни экстремальным, так как ей придется периодически очень сильно нагреваться, а затем остывать. Выдержать это сможет далеко не каждая даже современная смесь.

Современные технологии

Кроме того, не стоит упускать из виду экологичность штукатурки, чтобы в комнату не попадали при нагреве токсичные вещества. Также она должна быть эластичной и иметь хорошую теплопроводность. Цена готовой смеси очень и очень низкая.

Зачем штукатурить печь

Вопрос далеко непраздный, так как при изготовлении стараются сделать ее прочной и герметичной.

И, все-таки, встретить неоштукатуренную печь практически нереально, почему:

  1. Использование отделочного материала позволит украсить интерьер помещения и облагородить внешний вид сооружения.
  2. Глиняный раствор для штукатурки печи, используемый в виде штукатурки, является страховочным слоем, которые позволяет сделать печную кладку прочнее и герметичней. Связано это с объективными причинами. Так как перепады температуры кладка трескается, что может стать причиной попадания дыма в помещение. Это является негативным фактором не только для интерьера, но и для здоровья.

Если посмотреть на полки строительных магазинов, цена многих термостойких современных материалов превосходит традиционно применяемые смеси для оштукатуривания печей на основе глины.

Например, использовать можно глинопесчаный раствор с добавлением извести или цемента, подходит также раствор из извести и гипса.  Но, чаще всего стараются применять смеси на основе глины.

Заделка глиной кирпичной кладки

Совет: начинайте штукатурить кирпичную кладку только после полного ее застывания, тогда усадка не повлияет на результат. Обычно хватает 30 дней.

Знайте, что когда вы приготавливаете раствор для штукатурки печи – пропорции между компонентами глинопесчаной смеси зависят от жирности основного параметра, глины. Например, если она жирная – необходимо к 1 части глины добавить 4 части песка.

Совет: увеличить прочность штукатурного раствора поможет стекловолокно или асбест, которые необходимо добавить в готовую смесь.

Если же вам нужно приготовить раствор для штукатурки печной трубы, знайте, в нем обязательно должна быть гашеная известь. Иначе он отвалится от кирпичной кладки из-за конденсата.

Технология оштукатуривания печи

Ниже предлагается готовая инструкция к действию:

Подготовка

  1. Поверхность кирпичной кладки следует подготовить, очистив ее от грязи и остатков раствора (шпателем или щеткой по металлу), а также пыли (мягкой щеткой).

Совет: расчистите швы между кирпичами на глубину 5-10 мм, чтобы штукатурка лучше цеплялась к основанию.

  1. Забейте в стыки гвозди (l=40—50 мм) с шагом до 150 мм. Они должны выступать наружу примерно на 10 мм.
  2. Не поленитесь, обработайте поверхность кладки грунтовкой.
  3. Создать шероховатую поверхность и придать прочность раствору вам поможет также сетка из стекловолокна. Прикрепите ее к кладке жидкой смесью.

Совет: проводите оштукатуривание только горячих стенок.

Штукатурная металлическая сетка для раствора

Процесс

  1. Смочите кирпичную кладку водой. Раствор штукатурки наносите послойно.
  2. Первый слой сделайте жидким, консистенции сметаны.
  3. Наносите второй после отвердения первого, его толщина до 10 мм. Дождитесь, пока он схватится.
  4. Выровняйте поверхность. Для этого подождите, пока схватится раствор, и затрите все неровности на ней, предварительно смочив водой.

Совет: появившиеся после высыхания трещины расшейте, смочите водой, заполните раствором и, после его высыхания, затрите.

Требования к раствору

Печная штукатурка должна обладать особыми характеристиками, которые существенно отличаются от традиционных цементно-песчаных растворов, использующихся для отделки домов. Связано это с условиями ее работы – печная кладка расширяется при нагревании, поэтому крайне важно, чтобы готовая смесь была эластичной.

Приготовление раствора

Второй параметр – хорошая теплопроводность материала, чтобы печь быстро могла прогреть помещение.

Для этой цели в раствор добавляют разные компоненты:

  • глину;
  • асбест;
  • шамот;
  • соль;
  • стекловолокно.

Может применяться простая глина и сложные растворы на ее основе. Их можно приобрести в строительном магазине или сделать своими руками.

Таблица подготовки штукатурных смесей

Смеси для штукатурки печей

Каким раствором штукатурить печь – вопрос серьезный, так как от этого многое зависит. Компоненты в растворе могут иметь разное соотношение друг с другом в зависимости от жирности глины.

Ее жирность разбавляется песком, соответственно, чем выше данный параметр, тем больше песка понадобится. При смешивании вначале соединяются между собой сухие составляющие, куда затем добавляют глину или известь.

Совет: добавьте 200 г соли на ведро глинопесчаного раствора, чтобы увеличить его прочность.

Ниже предлагаются рецепты, как приготовить раствор для штукатурки печи самостоятельно:

  1. Первый состав раствора для штукатурки печей, который часто применяется для оштукатуривания поверхностей печей: 1:1:1/10:2 (глина, известь, асбест и песок). Компоненты смеси необходимо тщательно перемешать между собой, доводить ее следует до нужной кондиции водой, которую необходимо постепенно добавлять в емкость.
  1. Второй способ: 2:1:1 (песок, глина цемент М400 или М500). Вначале размешайте глину с водой, чтобы получилось густое тесто. Затем добавьте в раствор асбест, цемент и воду, тщательно перемешайте. Густота смеси должна напоминать крутую сметану. Недостаток – выработать раствор необходимо за 60 минут.

Консистенция готового раствора

Перед тем, как сделать раствор для штукатурки печи, вы должны понимать, что в любом составе есть – вода и вяжущий заполнитель, который может быть один или в смеси с другими компонентами, в частности, известь с цементом. При использовании высокопрочного гипсового раствора помните, что схватывается он в течение 6 минут, а спустя 30 минут он непригоден к использованию.

Вывод

Готовая смесь для штукатурки печей должна обладать особыми характеристиками, в частности, быть эластичной. Подготавливая раствор из рецептов, указанных выше, или из магазинных смесей необходимо это учитывать.

Приготовленный правильно раствор ровно и легко наносится на поверхность, а также без проблем заглаживается. Следите за вязкостью, которую регулируйте водой, не превышайте норму песка, иначе пластичность раствора пострадает. В представленном видео в этой статье вы найдете дополнительную информацию по данной теме.

Понравилась статья? Подписывайтесь на наш канал Яндекс.Дзен Добавить в избранное Версия для печати

Как приготовить цементный раствор | Статьи | Знания

Цементным раствором называют пластичную, однородно перемешанную смесь цемента и песка, затворённую определённым количеством воды. Процентное соотношение вяжущего и заполнителя рассчитывается исходя из области применения и требуемой марки раствора.

Основные свойства растворной смеси

Основными свойствами, которыми должен обладать правильно приготовленный раствор, считаются:

  1. Подвижность. Под этим понятием подразумевается способность готового состава растекаться под собственным весом. Степень подвижности определяется глубиной погружения в смесь эталонного конуса весом 300 г.
  2. Удобоукладываемость считается одной из основных характеристик и означает лёгкость укладывания и способность раствора растекаться тонким слоем на рабочую поверхность. Во многом зависит от водонасыщенности и подвижности состава.
  3. Водонасыщенность и водоудерживающие свойства цементного состава должны быть достаточными, чтобы не терять воду при укладке на пористые поверхности.
  4. Прочность на сжатие отвердевшего слоя зависит от марки используемого цемента. Во многом определяет сферу применения раствора.

Применяемые марки цемента и сферы их применения

В зависимости от прочности контрольного образца, в соответствии с ГОСТ 31108–2016, цемент разделяют на классы прочности. Но большинству строителей более привычны старые марки (М), поэтому приведём оба варианта маркировки и сферы их применения.

Класс (В)/Марка (М)Применение
7,5/100Внутренние работы по оштукатуриванию и выравниванию стен в сухих условиях.
15/200
22,5/300

Внутренние работы в умеренно влажных условиях, приготовление бетонов и растворов для подстилающих слоев.

32,5/400

Наиболее популярная марка цемента с широким применением, подходит практически для любых строительных, отделочных и ремонтных работ.

42,5/500Производство бетонных и ж/б изделий, применяется в сложных условиях эксплуатации.
52,5/600Производство монолитных и сборных ж/б конструкций повышенного уровня ответственности.

Определение марок цементного раствора и их применение

Надёжность кладки, штукатурного слоя или других цементных поверхностей напрямую зависит от правильно выбранной марки раствора, которая отличается от марки или класса используемого цемента. Для различных строительных работ используется смесь марок 15; 50; 75; 100; 150; 200.

Определение марки и пропорции

Марка получаемого раствора зависит от используемого цемента и количества добавленного песка, а также наличия специальных присадок, добавляемых для улучшения эксплуатационных характеристик раствора.

Наиболее удобно рассчитывать марку раствора не по классу, а по марке цемента. Приведём несколько наглядных примеров.

  • Чтобы получить раствор 100 марки, необходимо к одной части цемента М500 добавить 5 равных долей песка; к М400 – 4 части; а к М300 – 3. Соответственно пропорции составляют 1:5; 1:4 и 1:3.
  • Чтобы получить раствор марки 75, к цементу М500 необходимо примешать 6,5 части песка; к марке 400 – 5 частей; а к М300 – 4 (пропорции 1:6,5; 1:5 и 1:4).

Соответственно, для получения раствора более высокой марки, пропорция песка по отношению к цементу уменьшается.

Область применения различных марок раствора

Рассмотрим сферы применения основных марок раствора:

  • Растворы марки ниже М50 редко используются при проведении строительных работ из-за их недостаточной прочности. В основном ими заделываются горизонтальные швы при монтаже крупных блоков при возведении зданий складского назначения.
  • М50 обладает слабой прочностью, поэтому используется в основном при выравнивании незначительных неровностей перед оштукатуриванием и других подготовительных работах.
  • М75 подходит для кладки внутренних стен и перегородок без несущих функций, штукатурки и отделки сухих помещений.
  • М100 и М150 – наиболее применяющиеся марки раствора. Подходят для большинства строительных работ: кладки кирпичей или строительных блоков, проведения фасадного оштукатуривания, заливки или кладки фундаментов, выполнения стяжек, других целей.
  • М200 – раствор повышенной прочности, применяется при возведении мощных фундаментов, заливке полов с высокой проходимостью, строительства специальных объектов и в условиях агрессивного воздействия.

Виды цементного раствора

В зависимости от сферы применения выделяют следующие основные виды цементного раствора:

  1. Кладочный. Применяется для кладки любых штучных строительных материалов в надземных конструкциях, а также в агрессивных условиях (при увеличении марки до 200).
  2. Штукатурный. Используется при оштукатуривании стен монолитным слоем, а также нанесения штукатурки в два или более слоя.
  3. Монтажный. Этот вид предназначен для заполнения швов при сборке панельных или блочных конструкций.
  4. Специальные растворы получают путём добавления присадок, улучшающих водостойкость, теплоизоляционные, декоративные или другие свойства и характеристики, что расширяет их сферу применения.

Технология приготовления раствора для кладки

Приготовление раствора – мероприятие не сложное, но требующее соблюдения технологии и выдерживания пропорций. Для кладки стен и фасадной облицовки из кирпича и строительных блоков используется раствор марки 115, получаемый из соотношения 3,5:1 песка и цемента. Отмеривание ингредиентов в бытовых условиях проще всего производить вёдрами. Рассмотрим требования к каждому материалу, входящих в состав кладочного раствора:

  • Цемент должен быть свежим (смотрите дату изготовления) и без комков. Просроченный, слежавшийся цемент теряет свои свойства и не соответствует своей марке. Использовать его для ответственных конструкций нельзя.
  • Песок: мелкофракционный, чистый, без посторонних примесей, глины и грязи. Для облицовки фасада декоративным кирпичом рекомендуется просеять песок через мелкое сито.
  • Вода должна быть чистой. Не рекомендуется использовать дождевую воду или из технических ёмкостей, где могут быть остатки масел или другие загрязнения.

Для частного строительства или небольших ремонтных работ, раствор чаще всего замешивают в электрической мешалке или вручную. Рассмотрим оба метода.

Замешивание раствора в мешалке

Приготовление раствора в мешалке с электрическим приводом – наиболее простой, быстрый и лёгкий (в физическом плане) способ.

Весь процесс замеса можно разделить на следующие этапы (важно соблюдать очерёдность):

  1. В мешалку наливается вода, после чего она запускается. Важный нюанс, сколько необходимо воды. В среднем берётся ведро воды на ведро цемента, но может потребоваться больше, если песок слишком сухой. В мешалку заливается примерно ¾ от требуемого объёма.
  2. Следующий шаг – добавление моющего или жидкого мыла (не более 100 грамм на ведро воды) и перемешать в течение 2 минут, чтобы моющее растворилось полностью. Этот компонент необязателен, но при его добавлении работать с раствором становится приятнее.
  3. Теперь можно добавлять песок: примерно половину от требуемого количества. Это необходимо для того, чтобы смесь была жидкой на всём этапе перемешивания.
  4. Затем засыпается весь цемент, рассчитанный на один замес. Перемешивать около 5 минут.
  5. Заключительный шаг – добавляется вторая половина песка, после чего состав перемешивается ещё около 5 минут.

Как сделать раствор вручную

Если под рукой нет автоматической мешалки, раствор легко можно приготовить вручную в специальном металлическом рештаке, садовой тачке или на ровном участке. Для этого подойдёт бетонная плита, можно уложить на землю лист металла, сколотить деревянный щит или подстелить толстую полиэтиленовую плёнку, сложенную в несколько слоёв (самый крайний случай).

Сам процесс перемешивания ингредиентов отличается от работы с мешалкой. Порядок действий следующий:

  1. На ровную поверхность или в рештак кучно насыпается весь песок, отмерянный для одного замеса.
  2. На него аккуратно высыпается цемент, также весь от требуемого количества.
  3. Штыковой, совковой лопатой или при помощи тяпки перемешивается до однородного состояния (светло-серый цвет без видимых прослоек песка).
  4. Готовая смесь окучивается, в центре делается большое углубление, в которое наливается вода, примерно половина или меньше от требуемого объёма. Если решили добавить моющее, то оно вливается в заготовленную воду (до добавления в сухую смесь) и тщательно перемешивается.

  1. Сухой состав и воду нужно перемешивать аккуратно, в центре углубления, не давая возможности воде «убежать», так как с водой убежит часть цементного молочка, что негативно отразится на крепости раствора. По мере вымешивания воды, постепенно подливается новая порция.
  2. После того, как вся сухая смесь затворена водой, раствор тщательно вымешивается в течение 5–10 минут.

Ручной метод приготовления раствора более трудоёмок и занимает больше времени.

Добавки для цветного шва

Для выделения растворного шва в кирпичной или другой кладке, в процессе замешивания (на заключительном этапе), в состав добавляются красящие пигменты необходимого колера.

Вариантов может быть несколько:

  • Окрашивание раствора в один оттенок с лицевым кирпичом.
  • Придание шву контрастного оттенка.

Применение цветных пигментов имеет один недостаток: они имеют высокую стоимость, поэтому чаще всего цветным раствором проводят финишную затирку швов.

Для придания готовому раствору контрастного тёмного или даже чёрного цвета, во время приготовления раствора, в него можно добавить сажу или графитовую пыль. Но у этого способа есть минус – снижается марка раствора, поэтому изначально необходимо увеличить содержания цемента в составе.

Цементно-известковый раствор

Для оштукатуривания каменных, бетонных, кирпичных и других поверхностей внутри, а также снаружи помещений, применяется цементно-известковый раствор.

Он изготавливается по такой же технологии (перемешивание всех компонентов), но с учётом добавления извести. Наиболее часто используемые пропорции с использованием цемента М400 указаны в таблице.

ЦементИзвестьПесок
128
129
1211
1312
1315

Для оштукатуривания стен в сырых подвалах, а также цоколей, наружных стен и других участков, подверженных воздействию влаги, при приготовлении раствора в него вводятся гидроизоляционные присадки. Принцип действия большинства подобных присадок заключается в увеличении количества и скорости образования кристаллов эттрингита, которые в процессе твердения раствора уплотняют его структуру за счёт полного заполнения пор. Этот процесс, в свою очередь, приводит к увеличению водостойкости и морозостойкости отвердевшего раствора.

Особенности приготовления и применения раствора при отрицательных температурах

При необходимости проведения кладочных работ в холодное время года, в готовый раствор вводятся противоморозные добавки, которые повышают скорость гидратации и одновременно не позволяют воде, содержащейся в растворе замёрзнуть (до -10 °C). Противоморозные присадки бывают трёх типов:

  1. Воздухововлекающие.
  2. Пенообразующие.
  3. Комплексные.

При приготовлении раствора при отрицательных температурах особое внимание уделяется двум компонентам:

  1. Песку: он не должен быть замёрзшим, содержать комков и льда. Рекомендуется песок выдержать в отапливаемом помещении не менее суток.
  2. Воде: кроме добавления противоморозных добавок, рекомендуется использовать тёплую или даже горячую жидкость. Это позволит дополнительно прогреть песок, если в нём остались частички льда.

Применение такого кладочного раствора возможно при температуре воздуха до -15 °C, качество и прочность кладки при этом не страдает. При более низких температурах работы лучше отложить до потепления, так как потребуется создание тёплой завесы, что очень затратное мероприятие.

Технические стандарты

Классификация растворов и общие технические условия их производства регламентирует ГОСТ 28013–98. Технические требования, предъявляемые к цементу, который используется для приготовления растворов, содержит ГОСТ 25328–82. Более подробные и расширенные требования к приготовлению основных видов растворов собраны в СП 82–101–98.

как приготовить смесь, глиняный состав, пропорции для оштукатуривания, соотношение глины и песка, как сделать

Счастливых владельцев частных домов интересуют множество вопросов, так или иначе связанных со строительной тематикой. Если для жителей квартиры ремонт превращается в долговременный кошмар, способный значительно ухудшить качество жизни и потребовать значительных финансовых вложений. Для хозяина загородного дома ремонтные работы не прекращаются никогда, поэтому абсолютное большинство приобретают необходимые навыки, так сказать, в процессе.

Одним из важных вопросов, которые мы и рассмотрим в нашей статье, является выбор качественной смеси для оштукатуривания печи. Даже если у вас в доме автономное газовое или электрическое отопление, печные работы могут возникнуть при проектировании бани, сауны или даже камина, которым так любят украшать дома владельцы. Если вы только начали строительство, то здесь описаны размеры огнеупорного кирпича для кладки печи, по ссылке описано, как сделать раствор для кладки печи. Также вы можете ознакомиться с информацией о том, какой кирпич лучше для печи.

Состав

Покрытие поверхности печи отделочным слоем носит не только эстетический характер. Благодаря ему в помещение не будет проникать дым из возможных щелей в кладке печи, а также использование такого оборудования будет более безопасным и удобным.

Оптимальный раствор для штукатурки печи должен соответствовать следующим требованиям:

  • Не трескаться под воздействием температур.
  • Образовывать ровную однородную поверхность.
  • Являться безопасным для человека, не выделять наружу химических веществ.
  • Желателен максимально возможный срок беспроблемной эксплуатации.

Различают домашние (самодельные) и готовые смеси. Использование обоих типов имеет свои преимущества и недостатки. Для работы по оштукатуриванию поверхностей печи понадобится определенный строительный инструмент, а также минимальные навыки по выполнению подобных действий.

На видео – глиняный раствор для штукатурки печи:

Смесь для пользования своими руками

Большинство самодельных видов смеси готовятся на основе глины, песка и цемента. Использование химических пластификаторов и добавок должно быть исключительно подходящих марок, но обычно можно обойтись и вышеприведенными ингредиентами.

На основе бесценного строительного опыта множества домашних умельцев собрана подборка лучших рецептов приготовления таких смесей. Главное здесь — выдержать пропорции для раствора. Готовность смеси необходимо определять, что говорится «на глаз»: сметанообразная вязкая паста не должна быть слишком жидкой. Именно такая консистенция будет легче наноситься и обеспечит все необходимые характеристики готового покрытия. Вымешивать раствор лучше всего специальной насадкой «миксер» для дрели.

На видео рассказывается, как сделать раствор для штукатурки печи своими руками:

Рецепт № 1: глинянно – песчаная смесь

Традиционно используется глина в качестве главной составляющей. У каждого мастера могут немного видоизменяться соотношение и состав смеси. Классический рецепт, который можно взять за основу, приведен далее.

Необходимые ингредиенты:

  • 1 часть глины.
  • 2 части речного песка.
  • 0,1 части асбестового или стекловолокна.
  • Чистая вода.

В зависимости от жирности глины может потребоваться увеличить долю песка в составе. Вместо асбестового волокна, которое придает смеси пластичность и прочность, ранее использовали солому и измельченные части пеньки. Это, конечно же, более экологичный вариант. Весьма необычным способом дополнительного армирования будет использование небольших кусков мешковины, предварительно вымоченных в растворе.

Как сделать потолок из МДФ своими руками?

Здесь можно посмотреть фото панелей для наружной отделки дома.

Отзывы о фиброцементные панелейдля наружной отделки дома: https://resforbuild.ru/paneli/dlya-sten/fibrocementnye-dlya-naruzhnoj-otdelki-doma.html.

Рецепт № 2: цементно – глинопесчаная смесь

Обеспечить дополнительную твердость покрытия должен цемент в составе, поэтому часто используется и другой рецепт приготовления таких смесей. Перед формированием смеси очень важно убедиться в качестве цемента, ведь этот материал очень капризен при хранении, и даже малейшее попадание влаги способно безвозвратно погубить нужные характеристики.

Необходимые ингредиенты:

  • 1 часть портландцемента.
  • 1 часть глины.
  • 2 части речного песка.
  • 0,1 части асбестового или стекловолокна.
  • Чистая вода.

Глину для печного раствора необходимо тщательно просеять и замочить в воде примерно на ночь, чтобы она хорошо разбухла.

На видео – раствор для штукатурки домашней печи:

Рецепт № 3: известково – глинопесчаная смесь

В такие растворы часто добавляют и известь для придания дополнительной пластичности и визуальной привлекательности нанесенного покрытия. Этот материал выступает в качестве натурального пластификатора, но немного замедляет общее время сушки слоя.

Необходимые ингредиенты:

  • 1 часть гашеной извести.
  • 2 части речного песка.
  • 0,1 части асбестового или стекловолокна.
  • Чистая вода.

Песок перед вымешиванием раствора необходимо хорошо просеять от мусора и ненужных включений. Оптимально использовать речной песок, который по своим характеристикам прекрасно подходит для отделки печи.

Рецепт № 4: известково – гипсовая смесь

Характерной особенностью такой разновидности печной отделки будет исключительно быстрое застывание раствора. Это объясняется наличием гипса в составе, который полностью твердеет уже через полчаса после разведения с водой. Целесообразней будет использовать небольшое количество смеси за раз, чтобы избежать ненужных затрат и трудопотерь.

Необходимые ингредиенты:

  • 2 части гашеной извести.
  • 1 часть строительного гипса.
  • 1 часть речного песка.
  • 0,2 части асбестового или стекловолокна.
  • Чистая вода.

Приготовленный по этому рецепту состав обладает исключительной твердостью и как уже было сказано ранее, очень быстро сохнет, что также в определенных условиях играет только положительную роль для эксплуатации печного оборудования.

Важный нюанс: наносить слой штукатурки по любому из предложенных рецептов необходимо на чуть теплую печь, чтобы впоследствии поверхность не полопалась.

Стоит ли использовать готовые продукты

Традиционные рецепты, несмотря на хорошую проверку временем часто не дают желаемый результат. Причин может быть несколько, вплоть до несоблюдения температурного режима и низкого качества сырья. Не следует списывать со счетов и необходимую квалификацию, которую в наше время редко можно встретить. Раньше хороших печников знали в радиусе многих километров, но сейчас эта профессия практически исчезла.

Именно для трудных случаев, особенно когда попросту нет времени заниматься необходимым подбором материалов и изучению опытным путем всех предложенных рецептов, существует вариант с использованием специальных готовых смесей. Приобрести такие можно в строительном магазине, особенно поможет в этом консультация продавца. Готовые смеси для оштукатуривания печи — товар специфический, поэтому время поисков может затянуться.

На видео – пропорции для раствора для штукатурки печи:

Также при выполнении работ рекомендована предварительная обработка поверхности термостойкой грунтовкой, а для армирования можно использовать мелкоячеистую стальную сетку. Алгоритм действий и необходимые пропорции сухой смеси и воды указаны на упаковке и могут отличаться у разных производителей. А о том, какие есть огнеупорные материалы для печей читайте в нашей статье.

Также следует учесть, что стоимость готовых смесей априори выше самодельных растворов. Если этот момент не принципиален, вполне можно воспользоваться именно таким вариантом.

Оштукатуривание печи — дело весьма важное и ответственное. От качества верхнего слоя будет зависеть эстетический вид помещения, а также удобство использования печного оборудования. Сквозь небольшие щели и трещинки между кирпичами печи может просачиваться дым и вредные вещества, поэтому облицовка так важна для противопожарной безопасности и здоровья домочадцев.

Для штукатурки печи используются самодельные растворы, несколько рецептов которых приведены в нашей статье. Существуют также покупные смеси, в которых уже просчитаны необходимые пропорции, осталось только добавить воды и вымешать. Использование готовых смесей увеличит стоимость работ, но превосходный результат будет просто гарантирован. Возможно, вам также будет полезна информация о видах декоративных штукатурок для внутренней отделки. Отзывы о теплой штукатурке вы найдете тут. Также читайте о том, как сделать фактурную штукатурку из шпаклевки.

Как приготовить раствор для кладки печи: пропорции, состав, разновидности

Дата: 23 мая 2017

Просмотров: 4645

Коментариев: 1

Многие владельцы частных домов возводят печные конструкции из кирпича. Правильно выполненный замес влияет на качество кладки, герметичность конструкции, стойкость к повышенной температуре и ресурс эксплуатации печки. Важно знать, как приготовить раствор для кладки печи. Пропорции необходимо соблюдать – недостаток или повышенная концентрация ингредиентов вызывает появление трещин, нарушает целостность, способствует выходу угарного газа.

Глина – проверенный материал, позволяющий приготовить раствор для печи. Она обладает уникальными характеристиками. Глиняный раствор для кладки после высокотемпературного обжига по структуре и физическим характеристикам соответствует материалу печи – керамическому кирпичу, с которым образует единый массив. Недостаточно перемешать песок с глиной и добавить воду. Важно правильно выдержать рецептуру, соблюдать технологические рекомендации.

Рассмотрим, как избежать ошибок, производя приготовление раствора для кладки печей, остановимся на видах составов и технологии.

Глина — незаменимый материал при приготовлении кладочного раствора

Устройство печи и применяемые смеси

Чтобы подобрать правильный печной раствор для кладки разберемся с устройством печи, применяемыми при ее возведении смесями и материалами.

Конструкция состоит из следующих частей:

  • Основы фундамента. Она представляет железобетонный массив, не связанный с основанием здания.
  • Фундамента. Это нагруженная часть конструкции, которая воспринимает массу при незначительных тепловых нагрузках. Применяется сложная известково-цементная смесь, которая связывает красный полнотелый кирпич.
  • Теплоаккумулирующей основы. Температура во внутренней части составляет выше 500 °C. Устойчивость к воздействию агрессивных газов и кислотного конденсата обеспечивает цельный керамический кирпич, установленный на глиняный раствор для кладки.
  • Топки. Жаровая часть воспринимает умеренное химическое воздействие при нагреве, составляющем более 1000 °C. Топка сложена из шамотного кирпича, при установке которого использовалась огнеупорная смесь глины с добавлением шамота.
  • Дымохода. Нижняя часть дымохода, собранная из красного керамического кирпича на глиняном составе, воспринимает температуру до 400 °C. В центральной части дымохода используется печной кирпич. Применяется известковый раствор для печи.
  • Дымовой трубы. Прочность конструкции, воспринимающей ветровые нагрузки, обеспечивается благодаря применению красного кирпича, установленного на известковый состав.

Не существует строго определенного соотношения песка и глины для получения хорошего раствора

Какой раствор необходим для кладки печи

В процессе эксплуатации действуют механические нагрузки, связанные со следующими факторами:

  • усадкой конструкции;
  • ветровым давлением на трубу;
  • температурными перепадами.

Несмотря на достоинства цемента, подготовленный на его основе раствор для кладки печей, не может обеспечить целостность конструкции, воспринимая значительные перепады температуры и серьезные нагрузки.

Необходим состав, который является:

  1. Жаростойким. Такой состав сохраняет структуру при значительном нагреве с дальнейшим остыванием без возникновения трещин. Он в нагретом состоянии воспринимает статические усилия с сохранением несущей способности.
  2. Огнеупорным. Это жаростойкий материал, который воспринимает повышенную температуру, устойчив к агрессивному воздействию химических веществ, содержащихся в дымовых газах.

Указанным критериям в полной мере соответствует раствор для кладки печей, рецептура которого предусматривает использование глины, гашеной извести, шамота и портландцемента.

Глиняный раствор используется для основной конструкции печи, может использоваться также и для облицовки

Печной раствор для кладки – разновидности

Для правильной кладки различных участков следует использовать соответствующие составы:

  • глиняный. Это недорогой вариант, позволяющий использовать местные ресурсы. Состав обеспечивает необходимую прочность и жаростойкость на уровне 1100 °C. Материал устойчив к воздействию открытого огня и агрессивных газов. При добавлении шамота способен выдержать температуру до 1300 °C. Твердеет под воздействием повышенных температур. Подготовленная смесь может использоваться на протяжении неограниченного времени. При высыхании восстанавливает характеристики после смачивания водой;
  • известковый. Отличается повышенной ценой, так как предусматривает использование покупных материалов – известкового теста или негашеной извести. По прочностным характеристикам незначительно превышает глиняную смесь. Характеризуется средним уровнем огнеупорности. Параметры жаростойкости ограничиваются температурным диапазоном на уровне 400–500 °C. Устойчив к повышенной влажности. После замешивания может использоваться на протяжении трех суток.
  • цементный. Раствор для кладки печи на основе цемента дороже и прочнее, чем известковый. Применяется цемент М300. При необходимости, вводится известь или шамот. Цементно-известковый состав сохраняет свойства при температуре до 250 °C и может пропускать газы через поры. Цементно-шамотная смесь имеет повышенную до 1300 °C жаростойкость и не позволяет дымовым газам просачиваться. Подготовленной смесью следует воспользоваться в течение часа с момента замешивания.

Печи кладут на глиняный раствор, добавляя для прочности немного соли или цемента

Как подготовить раствор для кладки печи

Для сооружения печи пропорции вводимых ингредиентов определяют экспериментальным путем зависимо от жирности глины. Различают следующие виды:

  • Жирная – отличается пластичностью, но покрывается трещинами при испарении влаги.
  • Тощая – не пластична, имеет малую прочность, при высыхании крошится.
  • Нормальная – при твердении незначительно изменяется в объеме, пластична, не растрескивается.

Приготовление раствора для кладки печей на базе глины производите, соблюдая следующий алгоритм:

  • просейте песок на сите с ячейкой 1,5 мм;
  • промойте песок;
  • измельчите глину, насыпьте в бадью, залейте водой;
  • меняйте воду, перемешивая с глиной, до пастообразной консистенции и полной промывки;
  • добавьте песок в глину, соблюдая пропорцию 1:2;
  • перемешайте равномерно;
  • добавьте, при необходимости, для повышения прочности соль в количестве 0,1–0,25 кг и цемент (0,75 кг) на ведро смеси.

Песчано-цементный замес производите, смешивая песок с цементом в соотношении 2:1 или 3:1, добавьте воду до требуемой консистенции. Известковый состав готовится аналогичным образом. Гашеную известь смешивают с песком (1:2 или 1:3). На пропорцию влияет консистенция извести. Для увеличения прочности может добавляться асбест в объеме 10% от количества глины.

Приготовление огнеупорной смеси производится путем добавления шамота, который смешивается с глиной в равных отношениях. Добавляется вода в количестве 25% от массы глины. Ингредиенты перемешиваются до пластичной консистенции.

Желательно экспериментальным путем определить соотношение компонентов, а затем готовить полный объем. Определяя потребность в материалах, помните, что на 50 кирпичей понадобится два ведра кладочной смеси.

Заключение

Соблюдая приведенные рекомендации, несложно приготовить раствор для кладки печи. Важно использовать качественные составляющие, экспериментальным путем добиться необходимой консистенции, а также применять для различных частей соответствующие составы. Качество приготовленного состава гарантирует длительный ресурс эксплуатации.

На сайте: Автор и редактор статей на сайте pobetony.ru
Образование и опыт работы: Высшее техническое образование. Опыт работы на различных производствах и стройках — 12 лет, из них 8 лет — за рубежом.
Другие умения и навыки: Имеет 4-ю группу допуска по электробезопасности. Выполнение расчетов с использованием больших массивов данных.
Текущая занятость: Последние 4 года выступает в роли независимого консультанта в ряде строительных компаний.

Как приготовить раствор для оштукатуривания печи?

 

На сегодняшний день печь не играет такой важной роли, как это было еще сотню лет назад. Для отопления используются современные автономные котлы, снабженные продвинутыми системами электронного управления. Но они сильно зависят от поставки электричества и газа, а вот печь можно топить в любых условиях, были бы дрова.


Поэтому раствор для штукатурки печи нужно уметь готовить и в наши дни, ведь в своем доме никогда не помешает этот традиционный элемент русской культуры. При помощи дровяной печи можно не только отапливать дома, но и готовить очень вкусные блюда, которые ни при каких условиях не получатся в духовке. Многие хозяйки хорошо относятся к старославянской кухне, поэтому стараются повторять рецепты наших предков и сейчас.

Пропорции раствора


Сейчас уже только люди старшего поколения знают, как сделать раствор для штукатурки печи. Молодежь привыкла, что любые строительные материалы можно купить в специализированном гипермаркете, так что не нужно думать, как их там готовят. В состав обычно включают всего шесть основных компонентов, которые перемешивают до образования однородной массы.

 

 

От количества каждого вещества будет зависеть эксплуатационная пригодность раствора, так что нужно все тщательно выверять. Для создания рабочей смеси нужно будет подготовить следующие элементы:

 

  • •    термостойкую глину с максимальным показателем жирности;
  • •    обычный строительный цемент;
  • •    известь, измельченную до порошкового состояния;
  • •    гипс или мел;
  • •    кварцевый песок с мельчайшими фракциями;
  • •    асбест, который служит для адгезии и связи компонентов.


Пропорции раствора для штукатурки печи могут быть четырех типов. Выбор конкретного варианта зависит от материала, использованного при кладке и других сопутствующих моментов. Стоит заметить, что первые три компонента очень редко используются друг с другом, так как они взаимозаменяемы. Лишь отдельные случаи требуют использования сразу нескольких вяжущих веществ. А вот асбест входит в состав каждого раствора, потому что без него не получится добиться необходимого уровня прочности.

 


Можно добавлять в состав штукатурного раствора для печи и современные компоненты, которые помогут придать ему нужных свойств. Но перед этим нужно будет обязательно проверить свойства полученной смеси на небольшом участке. Лучше делать эксперименты с маленьким количеством раствора, чтобы не испортить много компонентов.

 

Большинство из них относится к категории дешевых, но все-таки большие лишние затраты никогда не приветствуются. Практически у каждого мастера в старину был свой собственный рецепт приготовления раствора, так что в ходе экспериментов можно найти отличный вариант.

Как готовить раствор для обмазывания печи?


После нахождения всех ингредиентов, нужно будет взять чистую емкость, которая будет служить для приготовления раствора для штукатурки печи. Но перед началом процедуры нужно пройти через подготовительный этап, который будет состоять из следующих шагов:

 

  • 1.    Все материалы должны состоять из мельчайших фракций и не иметь никаких крупных частиц, так как они будут серьезно мешать во время работы. Поэтому сырье просеивают через мелкое сито перед непосредственным смешиванием.
  • 2.    Вода должна быть комнатной температуры, примерно на уровне +20 градусов по Цельсию. Это оптимальный показатель для приготовления раствора.
  • 3.    Стоит учитывать, что чем больше цемента будет в составе, тем быстрее нужно работать со смесью, так как данный компонент уменьшает время застывания.
  • 4.    Перемешивать раствор нужно до образования однородного состава, в котором не будут присутствовать комки глины.
  • 5.    Очень важно не переусердствовать с водой, чтобы раствор не получился слишком жидким, так как тогда он попросту будет стекать со стенок печи.

 

 

 

Прочтение практических рекомендаций позволило узнать, как приготовить глиняный раствор для оштукатуривания печи. Теперь остается лишь использовать полученные навыки на практике. Никаких сложностей с этим возникнуть не должно, так как процесс предельно прост и понятен. А современные инструменты еще и помогут его упростить. Например, перемешивать штукатурку будет намного легче, если взять специальную насадку для дрели.

 


Если же есть желание использовать цементный раствор для штукатурки печи, то нужно готовить его небольшими порциями, так как израсходовать всю смесь нужно будет за 5-6 минут. По прошествии этого времени работать с ней уже будет очень сложно, а через полчаса она вообще становится непригодна к дальнейшей эксплуатации.

Раствор нужен для кладки плитки


Если же параллельно с оштукатуриванием пользователь желает положить плитку, то подойдет только цементный раствор. Глина не будет обладать достаточным вяжущим свойством, чтобы удерживать на себе еще и декоративные элементы, которые имеют довольно внушительный вес. Лучше добавить побольше асбеста, который укрепит раствор и придаст ему необходимые технические характеристики для наилучшего выполнения возложенных функциональных обязанностей.


Раствор для штукатурки печки под плитку сохнет очень быстро, так что нужно заранее подготовить все материалы, чтобы не пришлось потом в суматохе бегать и искать их, пока приготовленная для кладки основа приходит в негодность. Выбирая плитку, нужно обязательно убедиться в ее термостойких свойствах, так как печь будет постоянно нагреваться и остывать, поэтому материал должен обязательно выдерживать подобные циклы.

Глава 12.1: Подготовка растворов — Chemistry LibreTexts

  1. Последнее обновление
  2. Сохранить как PDF
  1. Приготовление растворов
    1. Пример 12.1.1
    2. Пример 12.1.2
    3. Пример 12.1.3
  2. Концентрации ионов в растворе
    1. Пример 12.1.4
    2. Ключевые уравнения
    3. Резюме
    4. Ключевые выводы
    5. Концептуальные проблемы
    6. Ответ
    7. Числовые задачи
    8. Ответы
    9. Участники

Цель обучения

  • Для количественного описания концентраций растворов.

В разделе 9.3 мы описали различные способы определения концентрации раствора, молярности (M), моляльности (m), процентной концентрации и мольной доли (X).Количество растворенного вещества, растворенного в определенном количестве растворителя или раствора. раствора описывает количество растворенного вещества, которое содержится в определенном количестве растворителя или раствора. Знание концентрации растворенных веществ важно для контроля стехиометрии реагентов для реакций, протекающих в растворе. В этом разделе описывается, как можно приготовить растворы из основного раствора известной концентрации

.

Приготовление растворов

Чтобы приготовить раствор, который содержит определенную концентрацию вещества, необходимо растворить желаемое количество молей растворенного вещества в достаточном количестве растворителя, чтобы получить желаемый конечный объем раствора.

\ (Молярность раствора = dfrac {моль \: of \: solute} {Объем раствора} \ tag {12.1.1} \)

Рисунок 12.1.1 иллюстрирует эту процедуру для раствора дигидрата хлорида кобальта (II) в этаноле. Обратите внимание, что объем растворителя не указан. Поскольку растворенное вещество занимает место в растворе, необходимый объем растворителя почти всегда на меньше , чем желаемый объем раствора. Например, если желаемый объем был 1,00 л, было бы неправильно добавлять 1.00 л воды на 342 г сахарозы, потому что это даст более 1,00 л раствора. Как показано на рисунке 12.1.2, для некоторых веществ этот эффект может быть значительным, особенно для концентрированных растворов.

Рисунок 12.1.1 Приготовление раствора известной концентрации с использованием твердого вещества

Рисунок 12.1.2 Приготовление 250 мл раствора (NH 4 ) 2 Cr 2 O 7 в воде

Растворенное вещество занимает пространство в растворе, поэтому для приготовления 250 мл раствора требуется менее 250 мл воды.

Пример 12.1.1

Раствор на рисунке 12.1.1 содержит 10,0 г дигидрата хлорида кобальта (II), CoCl 2 · 2H 2 O, в этаноле, достаточном для приготовления ровно 500 мл раствора. Какова молярная концентрация CoCl 2 · 2H 2 O?

Дано: масса растворенного вещества и объем раствора

Запрошено: концентрация (M)

Стратегия:

Чтобы найти количество молей CoCl 2 · 2H 2 O, разделите массу соединения на его молярную массу.Рассчитайте молярность раствора, разделив количество молей растворенного вещества на объем раствора в литрах.

Решение:

Молярная масса CoCl 2 · 2H 2 O составляет 165,87 г / моль. Следовательно,

\ (молей \: CoCl_2 \ cdot 2H_2O = \ left (\ dfrac {10.0 \: \ cancel {g}} {165 .87 \: \ cancel {g} / mol} \ right) = 0 .0603 \: mol \)

Объем раствора в литрах

\ (volume = 500 \: \ cancel {mL} \ left (\ dfrac {1 \: L} {1000 \: \ cancel {mL}} \ right) = 0.500 \: L \)

Молярность — это количество молей растворенного вещества на литр раствора, поэтому молярность раствора составляет

.

\ (молярность = \ dfrac {0,0603 \: mol} {0,500 \: L} = 0,121 \: M = CoCl_2 \ cdot H_2O \)

Упражнение

Раствор, показанный на рисунке 12.1.2, содержит 90,0 г (NH 4 ) 2 Cr 2 O 7 в достаточном количестве воды, чтобы получить конечный объем ровно 250 мл. Какова молярная концентрация дихромата аммония?

Ответ: (NH 4 ) 2 Cr 2 O 7 = 1.43 млн

Чтобы приготовить конкретный объем раствора, который содержит указанную концентрацию растворенного вещества, нам сначала нужно вычислить количество молей растворенного вещества в желаемом объеме раствора, используя соотношение, показанное в уравнении 12.1.1. Затем мы переводим количество молей растворенного вещества в соответствующую массу необходимого растворенного вещества. Эта процедура проиллюстрирована в Примере 12.1.2.

Пример 12.1.2

Так называемый раствор D5W, используемый для внутривенного восполнения биологических жидкостей, содержит 0.310 М. глюкозы. (D5W представляет собой примерно 5% раствор декстрозы [медицинское название глюкозы] в воде.) Рассчитайте массу глюкозы, необходимую для приготовления пакета D5W объемом 500 мл. Глюкоза имеет молярную массу 180,16 г / моль.

Дано: молярность, объем и молярная масса растворенного вещества

Запрошено: Масса растворенного вещества

Стратегия:

A Рассчитайте количество молей глюкозы в указанном объеме раствора, умножив объем раствора на его молярность.

B Получите необходимую массу глюкозы, умножив количество молей соединения на его молярную массу.

Решение:

A Сначала мы должны вычислить количество молей глюкозы, содержащихся в 500 мл 0,310 М раствора:

\ (V_L M_ {моль / л} = моль \)

\ (500 \: \ cancel {mL} \ left (\ dfrac {1 \: \ cancel {L}} {1000 \: \ cancel {mL}} \ right) \ left (\ dfrac {0 .310 \: моль \: глюкоза} {1 \: \ cancel {L}} \ right) = 0.155 \: моль \: глюкоза \)

B Затем мы переводим количество молей глюкозы в требуемую массу глюкозы:

\ (масса \: of \: глюкоза = 0,155 \: \ cancel {моль \: глюкоза} \ left (\ dfrac {180.16 \: g \: глюкоза} {1 \: \ cancel {моль \: глюкоза}} \ справа) = 27,9 \: г \: глюкоза \)

Упражнение

Другой раствор, обычно используемый для внутривенных инъекций, — это физиологический раствор, 0,16 М раствор хлорида натрия в воде. Рассчитайте массу хлорида натрия, необходимую для приготовления 250 мл физиологического раствора.

Ответ: 2,3 г NaCl

Раствор желаемой концентрации также можно приготовить путем разбавления небольшого объема более концентрированного раствора дополнительным растворителем. Базовый раствор — это коммерчески приготовленный раствор известной концентрации, который часто используется для этой цели. Разбавление основного раствора является предпочтительным, поскольку альтернативный метод взвешивания крошечных количеств растворенного вещества трудно осуществить с высокой степенью точности. Разбавление также используется для приготовления растворов из веществ, которые продаются в виде концентрированных водных растворов, таких как сильные кислоты.

Процедура приготовления раствора известной концентрации из основного раствора показана на рисунке 12.1.3. Это требует расчета желаемого количества молей растворенного вещества в конечном объеме более разбавленного раствора, а затем расчета объема исходного раствора, который содержит это количество растворенного вещества. Помните, что разбавление данного количества основного раствора растворителем не приводит к изменению числа , а не количества молей присутствующего растворенного вещества. Соотношение между объемом и концентрацией основного раствора и объемом и концентрацией желаемого разбавленного раствора, следовательно, составляет

\ ((V_s) (M_s) = моли \: of \: solute = (V_d) (M_d) \ tag {12.1.2} \)

, где нижние индексы s и d обозначают исходный и разбавленный растворы соответственно. Пример 5 демонстрирует расчеты, связанные с разбавлением концентрированного исходного раствора.

Рисунок 12.1.3 Приготовление раствора известной концентрации путем разбавления исходного раствора (a) Объем ( V s ), содержащий желаемые моли растворенного вещества (M s ), измеряют из исходного раствора. раствор известной концентрации.(b) Отмеренный объем исходного раствора переносят во вторую мерную колбу. (c) Измеренный объем во второй колбе затем разбавляется растворителем до объемной отметки [( V s ) (M s ) = ( V d ) (M d ). ].

Пример 12.1.3

Какой объем 3,00 М исходного раствора глюкозы необходим для приготовления 2500 мл раствора D5W в Примере 4?

Дано: Объем и молярность разбавленного раствора

Запрошено: объем основного раствора

Стратегия:

A Рассчитайте количество молей глюкозы в указанном объеме разбавленного раствора, умножив объем раствора на его молярность.

B Чтобы определить необходимый объем исходного раствора, разделите количество молей глюкозы на молярность исходного раствора.

Решение:

A Раствор D5W в Примере 4 содержал 0,310 М глюкозы. Начнем с использования уравнения 12.1.2 для расчета количества молей глюкозы, содержащихся в 2500 мл раствора:

\ (моль \: глюкоза = 2500 \: \ cancel {mL} \ left (\ dfrac {1 \: \ cancel {L}} {1000 \: \ cancel {mL}} \ right) \ left (\ dfrac { 0.310 \: моль \: глюкоза} {1 \: \ cancel {L}} \ right) = 0,775 \: моль \: глюкоза \)

B Теперь мы должны определить объем исходного раствора 3,00 M, который содержит это количество глюкозы:

\ (объем \: of \: stock \: soln = 0,775 \: \ cancel {mol \: gluosis} \ left (\ dfrac {1 \: L} {3 .00 \: \ cancel {mol \: глюкоза}} \ right) = 0,258 \: L \: или \: 258 \: mL \)

При определении необходимого объема исходного раствора мы должны были разделить желаемое количество молей глюкозы на концентрацию исходного раствора, чтобы получить соответствующие единицы.Кроме того, количество молей растворенного вещества в 258 мл исходного раствора такое же, как количество молей в 2500 мл более разбавленного раствора; изменилось только количество растворителя . Полученный нами ответ имеет смысл: разбавление основного раствора примерно в 10 раз увеличивает его объем примерно в 10 раз (258 мл → 2500 мл). Следовательно, концентрация растворенного вещества должна уменьшиться примерно в 10 раз, как это происходит (3,00 M → 0,310 M).

Мы также могли решить эту проблему за один шаг, решив уравнение 12.1,2 для В с и подставив соответствующие значения:

\ (V_s = \ dfrac {(V_d) (M_d)} {M_s} = \ dfrac {(2 .500 \: L) (0,310 \: \ cancel {M})} {3 .00 \: \ отменить {M}} = 0,258 \: L \)

Как мы уже отмечали, часто существует несколько правильных способов решения проблемы.

Упражнение

Какой объем 5,0 М маточного раствора NaCl необходим для приготовления 500 мл физиологического раствора (0,16 М NaCl)?

Ответ: 16 мл

Концентрации ионов в растворе

В разделе 9.3 мы подсчитали, что раствор, содержащий 90,00 г дихромата аммония в конечном объеме 250 мл, имеет концентрацию 1,43 М. {2-} (водн.) \ Tag {12.1.2} \)

Таким образом, 1 моль единиц формулы дихромата аммония растворяется в воде с образованием 1 моль анионов Cr 2 O 7 2- и 2 моль катионов NH 4 + (см. Рисунок 12.1.4).

Рисунок 12.1.4 Растворение 1 моля ионного соединения В этом случае растворение 1 моля (NH 4 ) 2 Cr 2 O 7 дает раствор, содержащий 1 моль Cr 2 O 7 2- ионов и 2 моль NH 4 + ионов.(Молекулы воды для ясности не показаны с молекулярной точки зрения.)

Когда мы проводим химическую реакцию с использованием раствора соли, такого как дихромат аммония, нам необходимо знать концентрацию каждого иона, присутствующего в растворе. Если раствор содержит 1,43 M (NH 4 ) 2 Cr 2 O 7 , то концентрация Cr 2 O 7 2- также должна быть 1,43 M, потому что существует один Cr 2 O 7 2- ионов на формульную единицу.Однако на формульную единицу приходится два иона NH 4 + , поэтому концентрация ионов NH 4 + составляет 2 × 1,43 М = 2,86 М. Поскольку каждая формульная единица (NH 4 ) 2 Cr 2 O 7 при растворении в воде образует три иона (2NH 4 + + 1Cr 2 O 7 2-), общая концентрация ионов в решение 3 × 1,43 M = 4,29 M.

Пример 12.1,4

Каковы концентрации всех веществ, полученных из растворенных веществ, в этих водных растворах?

  1. 0,21 М NaOH
  2. 3,7 M (CH 3 ) CHOH
  3. 0,032 M дюйм (NO 3 ) 3

Дано: молярность

Запрошено: концентрации

Стратегия:

A Классифицируйте каждое соединение как сильный электролит или как неэлектролит. — (водн.) \)

B Поскольку каждая формульная единица NaOH производит один ион Na + и один ион OH , концентрация каждого иона такая же, как концентрация NaOH: [Na + ] = 0.21 M и [OH ] = 0,21 M.

  • A Формула (CH 3 ) 2 CHOH представляет собой 2-пропанол (изопропиловый спирт) и содержит группу –OH, поэтому это спирт. Напомним из раздела 9.1, что спирты — это ковалентные соединения, которые растворяются в воде с образованием растворов нейтральных молекул. Таким образом, спирты не являются электролитами.

    B Таким образом, единственными растворенными веществами в растворе являются (CH 3 ) 2 молекул CHOH, поэтому [(CH 3 ) 2 CHOH] = 3.- (водн.) \)

    B Одна формульная единица In (NO 3 ) 3 дает один ион In 3 + и три иона NO 3 , поэтому 0,032 M In (NO 3 ) 3 раствор содержит 0,032 M In 3 + и 3 × 0,032 M = 0,096 M NO 3 , то есть [In 3 + ] = 0,032 M и [NO 3 ] = 0,096 М.

  • Упражнение

    Каковы концентрации всех веществ, полученных из растворенных веществ, в этих водных растворах?

    1. 0.0012 M Ba (OH) 2
    2. 0,17 M Na 2 SO 4
    3. 0,50 M (CH 3 ) 2 CO, широко известный как ацетон

    Ответ:

    1. [Ba 2 + ] = 0,0012 М; [OH ] = 0,0024 M
    2. [Na + ] = 0,34 М; [SO 4 2−] = 0,17 M
    3. [(CH 3 ) 2 CO] = 0,50 M

    Ключевые уравнения

    соотношение между объемом и концентрацией основного и разбавленного растворов

    Уравнение 12.1.2: \ ((V_s) (M_s) = моль \: of \: solute = (V_d) (M_d) \)

    Сводка

    Концентрация вещества — это количество растворенного вещества, присутствующего в данном количестве раствора. Концентрации обычно выражаются как молярность , количество молей растворенного вещества в 1 л раствора. Растворы известной концентрации могут быть приготовлены либо путем растворения известной массы растворенного вещества в растворителе и разбавления до желаемого конечного объема, либо путем разбавления соответствующего объема более концентрированного раствора (исходный раствор ) до желаемого конечного объема.

    Ключевые вынос

    • Концентрации раствора обычно выражаются в виде молярности и могут быть получены путем растворения известной массы растворенного вещества в растворителе или разбавления исходного раствора.

    Концептуальные проблемы

    1. Какое из представлений лучше всего соответствует 1 М водному раствору каждого соединения? Обоснуйте свои ответы.

      1. NH 3
      2. HF
      3. CH 3 CH 2 CH 2 OH
      4. Na 2 SO 4

    2. Какое из представлений, показанных в задаче 1, лучше всего соответствует 1 М водному раствору каждого соединения? Обоснуйте свои ответы.

      1. CH 3 CO 2 H
      2. NaCl
      3. Na 2 S
      4. Na 3 PO 4
      5. ацетальдегид
    3. Можно ли ожидать, что 1,0 М раствор CaCl 2 будет лучше проводить электричество, чем 1,0 М раствор NaCl? Почему или почему нет?

    4. Альтернативный способ определения концентрации раствора — моляльность , сокращенно м .Моляльность определяется как количество молей растворенного вещества в 1 кг растворителя . Чем это отличается от молярности? Ожидаете ли вы, что 1 M раствор сахарозы будет более или менее концентрированным, чем 1 m раствор сахарозы? Поясните свой ответ.

    5. Каковы преимущества использования решений для количественных расчетов?

    Ответ

    1. Если количество вещества, необходимое для реакции, слишком мало для точного взвешивания, использование раствора вещества, в котором растворенное вещество диспергировано в гораздо большей массе растворителя, позволяет химикам измерить количество вещества. вещество, точнее.

    Числовые задачи

    1. Рассчитайте количество граммов растворенного вещества в 1.000 л каждого раствора.

      1. 0,2593 M NaBrO 3
      2. 1,592 М ННО 3
      3. 1,559 М уксусная кислота
      4. 0,943 M йодат калия
    2. Рассчитайте количество граммов растворенного вещества в 1.000 л каждого раствора.

      1. 0.1065 Мбай 2
      2. 1,135 M Na 2 SO 4
      3. 1,428 M NH 4 Br
      4. 0,889 М ацетат натрия
    3. Если все растворы содержат одно и то же растворенное вещество, какой раствор содержит большую массу растворенного вещества?

      1. 1,40 л 0,334 М раствора или 1,10 л 0,420 М раствора
      2. 25,0 мл 0,134 М раствора или 10,0 мл 0,295 М раствора
      3. 250 мл 0.489 М раствор или 150 мл 0,769 М раствора
    4. Заполните следующую таблицу для 500 мл раствора.

      Соединение Масса (г) Моль Концентрация (м)
      сульфат кальция 4,86 ​​
      уксусная кислота 3.62
      дигидрат иодистого водорода 1,273
      бромид бария 3,92
      глюкоза 0,983
      ацетат натрия 2.42
    5. Какая концентрация каждого вида присутствует в следующих водных растворах?

      1. 0,489 моль NiSO 4 в 600 мл раствора
      2. 1,045 моль бромида магния в 500 мл раствора
      3. 0,146 моль глюкозы в 800 мл раствора
      4. 0,479 моль CeCl 3 в 700 мл раствора
    6. Какая концентрация каждого вида присутствует в следующих водных растворах?

      1. 0.324 моль K 2 MoO 4 в 250 мл раствора
      2. 0,528 моль формиата калия в 300 мл раствора
      3. 0,477 моль KClO 3 в 900 мл раствора
      4. 0,378 моль йодида калия в 750 мл раствора
    7. Какова молярная концентрация каждого раствора?

      1. 8,7 г бромида кальция в 250 мл раствора
      2. 9,8 г сульфата лития в 300 мл раствора
      3. 12.4 г сахарозы (C 12 H 22 O 11 ) в 750 мл раствора
      4. 14,2 г гексагидрата нитрата железа (III) в 300 мл раствора
    8. Какова молярная концентрация каждого раствора?

      1. 12,8 г гидросульфата натрия в 400 мл раствора
      2. 7,5 г гидрофосфата калия в 250 мл раствора
      3. 11,4 г хлорида бария в 350 мл раствора
      4. 4.3 г винной кислоты (C 4 H 6 O 6 ) в 250 мл раствора
    9. Приведите концентрацию каждого реагента в следующих уравнениях, принимая 20,0 г каждого реагента и объем раствора 250 мл для каждого реагента.

      1. BaCl 2 (водн.) + Na 2 SO 4 (водн.) →
      2. Ca (OH) 2 (водн.) + H 3 PO 4 (водн.) →
      3. Al (NO 3 ) 3 (водн.) + H 2 SO 4 (водн.) →
      4. Pb (NO 3 ) 2 (водн.) + CuSO 4 (водн.) →
      5. Al (CH 3 CO 2 ) 3 (водн.) + NaOH (водн.) →
    10. На эксперимент потребовалось 200.0 мл 0,330 М раствора Na 2 CrO 4 . Для приготовления этого раствора использовали исходный раствор Na 2 CrO 4 , содержащий 20,0% растворенного вещества по массе с плотностью 1,19 г / см 3 . Опишите, как приготовить 200,0 мл 0,330 М раствора Na 2 CrO 4 с использованием основного раствора.

    11. Гипохлорит кальция [Ca (OCl) 2 ] — эффективное дезинфицирующее средство для одежды и постельного белья.Если в растворе концентрация Ca (OCl) 2 составляет 3,4 г на 100 мл раствора, какова молярность гипохлорита?

    12. Фенол (C 6 H 5 OH) часто используется в качестве антисептика в жидкостях для полоскания рта и леденцах для горла. Если в жидкости для полоскания рта концентрация фенола составляет 1,5 г на 100 мл раствора, какова молярность фенола?

    13. Если таблетка, содержащая 100 мг кофеина (C 8 H 10 N 4 O 2 ), растворяется в воде с получением 10.0 унций раствора, какова молярная концентрация кофеина в растворе?

    14. На этикетке с определенным лекарством есть инструкция по добавлению 10,0 мл стерильной воды, в которой говорится, что каждый миллилитр полученного раствора будет содержать 0,500 г лекарства. Если пациенту назначена доза 900,0 мг, сколько миллилитров раствора следует ввести?

    ответы

    1. 1.74 × 10 −3 M кофеин

    Авторы

    Изменено Джошуа Халперн, Скотт Синекс и Скотт Джонсон

    Подготовка растворов | Химия для неосновных

    Цели обучения

    • Опишите процесс приготовления растворов.

    Сколько воды мне добавить?

    Передняя обложка кулинарной книги Crisco 1912 года, любезно предоставлено Викимедиа.

    Еще в «старые добрые времена» (когда бы это ни было на самом деле) многие повара не беспокоились о точных измерениях. Они «просто знали», сколько муки использовать или сколько добавить воды. Большинству из нас нужны более точные способы измерения, когда мы готовим. Химики очень осторожны при приготовлении растворов, поскольку результаты их экспериментов должны быть количественными. «Просто знать» недостаточно для научных целей.

    Подготовка растворов

    Если вы пытаетесь подготовить 1.00 л 1,00 раствора NaCl, вы получите 58,44 г хлорида натрия. Однако нельзя просто добавить хлорид натрия в 1,00 л воды. После растворения растворенного вещества объем раствора будет немного больше литра, поскольку гидратированные ионы натрия и хлорида занимают место в растворе. Вместо этого необходимо использовать мерную колбу . Мерные колбы бывают разных размеров (см. Изображение ниже) и предназначены для того, чтобы химик мог приготовить раствор только одного определенного объема.

    Другими словами, вы не можете использовать мерную колбу объемом 1 литр для приготовления 500 мл раствора. Его можно использовать только для приготовления 1 литра раствора. Действия, которые необходимо выполнить при приготовлении раствора с использованием мерной колбы объемом 1 литр, изложены ниже и показаны на рисунке ниже.

    1. Отвешивают соответствующую массу растворенного вещества и добавляют в мерную колбу, которая примерно наполовину заполнена дистиллированной водой.
    2. Раствор перемешивают до полного растворения растворенного вещества.
    3. Осторожно доливают дистиллированную воду до отметки на горлышке колбы.
    4. Колбу закрывают крышкой и несколько раз переворачивают до полного перемешивания.

    Рис. 1. Действия, которые необходимо выполнить при приготовлении раствора известной молярности: (A) взвесить правильную массу растворенного вещества, (B) растворить в растворителе в мерной колбе, и (C) добавить растворитель в линию наполнения на колбе и смешивание. От фонда CK-12 — Кристофер Ауён.

    Сводка

    • Описаны этапы приготовления раствора.

    Практика

    Посмотрите видео по ссылке ниже и ответьте на следующие вопросы:

    1. Какое решение?
    2. Что такое растворенное вещество?
    3. Что такое растворитель?
    4. Какие меры предосторожности необходимо соблюдать?

    Обзор

    1. Почему вы добавляете растворенное вещество в колбу, только частично заполненную растворителем?
    2. Сколько растворителя добавить в конце?
    3. Почему вы переворачиваете колбу несколько раз после разбавления до метки?

    Глоссарий

    • мерная колба: Калиброванная колба с длинным горлышком для точного приготовления растворов.

    Как приготовить раствор из маточного раствора

    Стандартный или стандартный раствор — это раствор, в котором вы точно знаете его концентрацию. Базовые растворы можно делать в химической лаборатории или покупать у производителей химикатов. Когда у вас есть основной раствор, вы можете приготовить растворы с более низкой концентрацией, разбавив концентрированный основной раствор.

    Разбавить означает добавить определенное количество растворителя (воды) к определенному количеству концентрированного основного раствора.Если вы добавите определенное количество растворителя к определенному количеству концентрированного исходного раствора, вы заметите, что количество растворенного вещества, присутствующего в исходном растворе, равно количеству, присутствующему в разбавленном растворе. Единственное отличие состоит в том, что разбавленный раствор теперь содержит больше воды, чем исходный раствор, из которого он был приготовлен. В реальной жизни то, что вы только что прочитали, похоже на то, как вы добавляете больше воды в кофе или чай, чтобы облегчить его вкус. По мере добавления воды вы увеличиваете только количество воды в растворе, но не количество присутствующих в нем молекул кофе или чая.Вот иллюстрация разбавленного раствора, приготовленного из основного раствора:

    Из приведенного выше рисунка видно, что в концентрированном растворе есть 7 молекул растворенного вещества и 7 молекул в разбавленном растворе. Но в разбавленном растворе больше растворителя, чем в концентрированном.

    Поскольку оба раствора содержат равное количество молекул растворенного вещества, отсюда следует, что:

    • количество молей химических веществ, присутствующих в концентрированном растворе, равно количеству молей химических веществ, присутствующих в разбавленном растворе

    Если мы переведем предыдущее утверждение в математическое выражение, мы получим:

    моль концентрированного раствора = моль разбавленного раствора

    Но поскольку мы знаем, что количество кофейного раствора в молях равно объему раствора, умноженному на его концентрацию в молярности, то мы можем записать, что:

    Объем (V) концентрированного раствора, умноженный на молярность (M) концентрированного раствора, равен объему (V) разбавленного раствора, умноженному на молярность (M) разбавленного раствора.Если перевести это в математическое выражение, мы получим:

    Расчет концентрации разбавленного раствора

    Как вы могли заметить, полученная нами формула является общей формулой для разбавления концентрированного раствора до раствора с более низкой концентрацией. Обратите внимание, что если вы указываете концентрацию раствора как 2 M , прописные буквы M часто используются для обозначения единицы (моль / л), которая является единицей измерения концентрации, указанной в молярности.

    Чтобы использовать формулу разбавления, вы должны знать как минимум три из четырех переменных в ней. Теперь давайте используем его, чтобы ответить на следующий вопрос:

    Предположим, что в одном из ваших общих химических экспериментов вам необходимо использовать 2 М серную кислоту (H 2 SO 4 ) для определенной реакции, но инструктор лаборатории дал вам 5 MH 2 SO 4 .Как бы вы это сделали?

    Решение

    Поскольку из вопроса вы знаете нужную вам концентрацию серной кислоты, значит, вы должны определиться с тем, какой объем вам нужен. Итак, допустим, вам нужно 10 мл 2 M H 2 SO 4 . Из этого следует, что вы должны рассчитать объем исходного раствора, который необходимо внести в пипетку, чтобы подготовить желаемую концентрацию. Для этого необходимо вспомнить формулу разведения. Эта формула говорит, что:

    Теперь, если вы прочитаете вопрос еще раз, вы заметите, что все единицы соответствуют тому, что нам нужно вычислить.По этой причине нет необходимости конвертировать единицы измерения. Следовательно:

    Vol con =? (Нам нужно посчитать, сколько нам нужно запасов)

    Vol dil = 10 мл (это объем необходимой вам концентрации 2 M)

    M con = 5 M (это молярность концентрированного раствора)

    M dil = 2 M (это молярность разбавленного раствора)

    Если подставить вышеуказанную информацию в формулу разведения, мы получим

    Поскольку нам нужен объем концентрированного основного раствора (Vol con ), мы должны разделить левую и правую части знака равенства в приведенном выше уравнении (1) на 5 M.Если сделаем, то получим:

    Исходя из расчета, вам нужно добавить пипеткой 4 мл 5 М раствора серной кислоты, чтобы приготовить 10 мл 2 М раствора серной кислоты. Чтобы приготовить 10 мл 2 М раствора, сначала необходимо перелить около 5 мл дистиллированной воды в мерную колбу на 10 мл. Затем медленно добавьте 4 мл исходного раствора (серной кислоты). Прокрутите колбу и долейте дистиллированной воды до отметки 10 мл.

    Внимание!

    Как вы могли заметить, мы добавили серную кислоту в дистиллированную воду в мерной колбе, а не наоборот. Почему?

    Мы сделали это для того, чтобы раствор не взорвался. Как вы, возможно, знаете, серная кислота намного плотнее воды, поэтому, когда вы добавляете ее в воду, ее молекулы могут перемещаться внутри и хорошо смешиваться с молекулами воды. Однако, поскольку вода менее плотная, чем серная кислота, если вы добавите ее к серной кислоте, ее молекулы создадут барьер, в котором вода будет наверху, а серная кислота — внизу. Этот нежелательный барьер может вызвать взрывную реакцию двух химикатов (экзотермическая реакция), генерирующая достаточно энергии, чтобы разбить колбу.Другим способом предотвратить эту реакцию можно, поместив мерную колбу в ледяную баню и добавив кислоту в дистиллированную воду. Лед поглотит энергию, вырабатываемую раствором, в результате чего раствор остынет.

    Чтобы узнать, как приготовить раствор из твердого вещества, щелкните здесь.

    Как приготовить химический раствор с нуля

    posted on

    Как приготовить химический раствор с нуля

    Для приготовления химического раствора вам понадобятся как минимум две вещи:

    Растворенное вещество — это химическое вещество в меньшем количестве, а растворитель — это химическое вещество в большем количестве.Растворенное вещество обычно растворяется в растворителе. Растворенное вещество или растворитель может быть твердым, жидким или газообразным.

    Например, чтобы приготовить раствор хлорида натрия (NaCl) с нуля, вы должны сначала решить, какую концентрацию хлорида натрия вы хотите. Поскольку мы будем растворять NaCl в воде, мы будем называть воду нашим растворителем, а NaCl — растворенным веществом.

    Например,

    Чтобы приготовить 500 мл 5,0 М раствора NaCl с нуля , необходимо

    нужно сделать следующее:

    • рассчитайте количество NaCl, необходимое для взвешивания

    Чтобы рассчитать количество NaCl, которое вам необходимо взвесить, необходимо сначала преобразовать 500 мл в литры, затем в моль, а затем в граммы.

    • Чтобы перейти от мл к литрам, вам понадобится коэффициент преобразования (1 л / 1000 мл), который получается из соотношения 1000 мл = 1 л.
    • Чтобы перейти от литров к молям, вам понадобится коэффициент преобразования (5 моль / л), который совпадает с концентрацией раствора, который вы готовите: 5M (5 моль / л).
    • Чтобы перейти от молей к грамму, вам понадобится коэффициент преобразования (58,44 г / моль), который совпадает с молярной массой химического вещества, которое вы используете для приготовления раствора.В этом примере это химическое вещество: NaCl.
    • Ниже приведен полный расчет количества NaCl, которое необходимо взвесить, чтобы приготовить 500 мл раствора NaCl с концентрацией 5 М.

    Ответ

    5,0 М раствор NaCl означает, что в 1 литре раствора NaCl присутствует 5,0 моль NaCl.

    Поскольку молярность указывается в молях на литр, мы должны преобразовать 500 мл в литры (л). Поскольку 1000 мл = 1 л, мы можем настроить его так, чтобы миллилитры (мл) компенсировали друг друга, литры (л) компенсировали друг друга и, наконец, моли компенсировали друг друга.Если мы это сделаем, общая настройка будет выглядеть так:

    Как рассчитать 5 M NaCl в граммах

    Чтобы приготовить 500,0 мл 5,0 M раствора NaCl,

    • отвешивают 146,1 г NaCl в мерную колбу.
    • Добавьте достаточное количество воды и взбалтывайте, пока весь NaCl не растворится.
    • Долейте еще воды до отметки 500 мл на мерной колбе.

    После этого вы поймете, что можете использовать приведенное выше математическое выражение для расчета количества любого химического вещества, которое необходимо взвесить для приготовления химического раствора.

    После расчета, вот иллюстрация, показывающая, как можно использовать мерную колбу для приготовления химического раствора известной концентрации, массы и объема:

    Как приготовить химический раствор

    Что такое раствор?

    Раствор представляет собой однородную (гомогенную) смесь растворенного вещества и растворителя. Раствор может существовать в виде газа, жидкости или твердого вещества.

    Чтобы узнать, как рассчитать концентрацию раствора, щелкните здесь, а как приготовить раствор из запаса, щелкните здесь.

    Как приготовить раствор правильной концентрации

    Обзор:

    Когда они находятся в химической лаборатории или просто пишут тест, многие студенты борются с тем, как создать раствор известной концентрации. Студентам это сложно, потому что они пытаются вычислить раствор в молях, а это единица измерения, которую невозможно измерить в лаборатории. Есть несколько простых шагов для приготовления подходящего раствора с известной концентрацией.

    Шаг 1. Определение количества молей соединения

    Первый шаг — определить, какую концентрацию раствора вам нужно приготовить вместе с желаемым объемом.В этом примере мы собираемся приготовить 1234 мл 1,54-молярного раствора NaCl (хлорид натрия).

    Сначала мы определим количество молей NaCl, содержащихся в 1234 мл 1,54 молярного раствора. Для этого воспользуемся следующей формулой:

    Где:

    n = количество молей раствора в молях

    C = концентрация раствора в моль / л

    V = объем раствора в л

    ПРИМЕЧАНИЕ: Всегда проверяйте соответствие единиц измерения.

    Обратите внимание, что единицей объема является мл, а единицей измерения C — моль / л. Это означает, что перед решением уравнения нам нужно преобразовать мл в L. 1234 мл — это то же самое при 1,234 л. Теперь, когда все наши единицы согласованы, мы можем заменить наши известные значения для концентрации и объема:

    Шаг 2: Определение массы соединения

    Теперь мы знаем, что нам нужно добавить в наш раствор 1,90 моль NaCl; однако моль — это не единица, которую можно измерить с помощью обычных лабораторных инструментов.Сначала мы должны преобразовать его в единицы, которые легко измерить в лаборатории. Для этого требуется новое уравнение:

    Где:

    n = количество молей в молях

    m = граммы соединения в г

    M = молярная масса соединения в г / моль (взято непосредственно из периодической таблицы)

    Чтобы определить M, нам нужно поближе взглянуть на наше соединение:

    NaCl = 1 Na + 1 Cl

    = (1 * 22,99 г / моль) + (1 * 35,45 г / моль)

    = 58.44 г / моль

    Мы определили нашу молярную массу (M) и вычислили n на шаге 1. Теперь мы готовы использовать замену, чтобы найти m. Поскольку все наши единицы согласованы, никаких дополнительных настроек не требуется.

    м = n * M

    Теперь мы знаем, что нам нужно отмерить 111,0 граммов NaCl на весах, чтобы добавить его в раствор.

    ПРИМЕЧАНИЕ: Ключевым моментом при создании любого раствора является медленное добавление соединения к воде при перемешивании.Это начнется с очень низкой концентрации соединения и будет постепенно увеличиваться до желаемой концентрации. Если вместо этого к соединению добавить воду, это начнется с очень высокой концентрации раствора, что потенциально может привести к образованию опасного соединения.

    Заполните химический стакан примерно на деионизированной водой, затем добавьте 111,0 г NaCl. После добавления NaCl заполните контейнер до отметки 1,234 л. Теперь у вас есть 1,234 л 1,54-молярного раствора NaCl.

    Заинтересованы в услугах репетиторства по химии? Узнайте больше о том, как мы помогаем тысячам студентов каждый учебный год.

    SchoolTutoring Academy — ведущая компания в сфере образовательных услуг для школьников и школьников. Мы предлагаем учебные программы для учащихся K-12, AP и колледжей. Чтобы узнать больше о том, как мы помогаем родителям и ученикам в Норфолке, штат Вирджиния, посетите раздел Репетиторство в Норфолке, штат Вирджиния

    .

    Как приготовить стандартный раствор?

    Как приготовить стандартный раствор?

    Приготовление стандартного раствора методом взвешивания
    • Раствор, концентрация которого точно известна, называется стандартным раствором .
    • Стандартный раствор можно приготовить методом взвешивания следующим образом.
      (a) Необходимая масса растворенного вещества рассчитывается и взвешивается.
      (b) Растворенное вещество растворяют в небольшом количестве дистиллированной воды в химическом стакане.
      (c) Раствор переносят в мерную колбу.
      (d) Добавляют еще дистиллированную воду для получения необходимого объема. Колбу закрывают пробкой и встряхивают.
    • Например, чтобы приготовить 1,0 дм 3 из 0,50 моль дм -3 водного гидроксида натрия.

    • Очень точный стандартный щелочной раствор не может быть приготовлен с использованием гидроксида натрия в качестве основного стандарта. Это связано с тем, что гидроксид натрия
      (а) обладает текучестью. Он впитывает влагу и растворяется с образованием раствора.
      (b) гидроксид натрия не может быть чистым. Реагирует с углекислым газом из воздуха:
      2NaOH (s) + CO 2 (g) → Na 2 CO 3 (s) + H 2 O (l)
      Следовательно, концентрация этого щелочной раствор не точен.
    • Чистый безводный карбонат натрия, Na 2 CO 3 , используется для приготовления первичного стандартного щелочного раствора.
    • Твердые органические кислоты, такие как щавелевая кислота, H 2 C 2 O 4 .2H 2 O, используются для приготовления первичного стандартного кислотного раствора.
    • Стандартный раствор также можно приготовить методом разбавления.

    Люди тоже спрашивают

    Приготовление стандартного раствора методом разбавления
    • Стандартный раствор можно также приготовить путем разбавления .Стандартные кислоты, такие как хлористоводородная кислота, серная кислота и азотная кислота, все получают путем разбавления коммерческих концентрированных кислот (исходных растворов) различными количествами дистиллированной воды.
    • Добавление воды в концентрированный раствор:
      (a) изменяет концентрацию раствора
      (b) не меняет количество молей присутствующего растворенного вещества
    • Это означает, что общее количество молей растворенных частиц до разбавления равно общему количеству молей растворенных частиц после выполнения разбавления.
      Моль растворенного вещества до разбавления = моль растворенного вещества после разбавления
    • Уравнение разбавления может быть получено, как показано ниже.


    Поскольку добавление воды не меняет количество молей растворенных частиц,
    M 1 V 1 = M 2 V 2

    На рисунке показано, как пример выполняется в лаборатории.

    • Требуемый объем основного раствора должен быть точно отмерен.Используйте пипетку , чтобы точно отмерить 25,0 см 3 основного раствора. Используйте бюретку для измерения любых других объемов, меньших 50,0 см 3 .
    • Например, чтобы приготовить 500 см 3 0,15 моль-дм -3 серной кислоты из исходного раствора 2,0 моль-дм -3 серной кислоты.
      (a) Рассчитайте необходимый объем основного раствора.

      (b) Заполните бюретку основным раствором.
      Закончился 37.5 см 3 маточного раствора в мерную колбу объемом 500 см 3 , содержащую немного дистиллированной воды.

      (c) Добавьте в колбу еще дистиллированной воды, постоянно помешивая. Затем с помощью пипетки добавьте дистиллированную воду, чтобы довести мениск до калибровочной отметки. Закройте колбу и хорошо встряхните, чтобы обеспечить тщательное перемешивание.

    1. Рассчитайте объем концентрированной азотной кислоты, 18 моль дм -3 , который требуется для приготовления 5.0 дм 3 2,0 моль дм -3 азотная кислота.
    Решение:

    2. Какой объем 2,0 моль дм -3 серной кислоты необходим для приготовления 100 см 3 0,5 моль дм -3 серной кислоты?
    Решение:

    Как приготовить 0,2 М раствор NaOH объемом 200 мл из 0,12 М раствора NaOH объемом 200 мл?

    Проблемы, требующие приготовления раствора с определенной молярностью и объемом, начиная с исходного раствора, всегда можно решить, используя уравнение для расчета разбавления.

    Когда вы разбавляете раствор, вы по существу увеличиваете объем раствора, сохраняя количество молей растворенного вещества постоянным и на увеличивая количество присутствующего растворителя.

    Поскольку молярность определяется как количество молей растворенного вещества на литр раствора, если вы увеличиваете объем без изменения количества молей растворенного вещества, молярность, конечно, будет уменьшаться с до .

    Это означает, что вы успешно разбавили исходный раствор, т.е.е. вы сделали его менее концентрированным .

    Однако обратите внимание, что ваш целевой раствор на самом деле более концентрированный , чем основной раствор — это несовместимо с идеей о том, что мы можем разбавить исходный раствор, чтобы получить целевой раствор.

    Итак, вы знаете, что ваш целевой раствор имеет объем 200 мл и молярность 0,2 ​​M . Это означает, что он должен содержать

    #C = n / V => n = C * V #

    #n_ (NaOH) = 0.(-3) cancel («L») = «0,04 моль» ## NaOH #

    Теперь вам нужно выяснить, как в каком объеме основного раствора будет содержаться такое количество молей гидроксида натрия.

    #C = n / V => V = n / C #

    #V_ «stock» = (0,04 отмена («моль»)) / (0,12 отмена («моль») / «L») = «0,333 л» = цвет (красный) («333 мл») #

    Вот в чем проблема с данными значениями. Чтобы получить такое количество молей гидроксида натрия из исходного раствора, вам понадобится 333 мл . Однако у вас есть только 200 мл .(-3) cancel («L») = «0,024 моль» ## NaOH #

    Как оказалось, у вас недостаточно молей растворенного вещества, чтобы приготовить целевой раствор из исходного раствора.

    Единственный способ приготовить целевой раствор и по-прежнему использовать этот исходный раствор — это растворить еще ед. Гидроксида натрия в исходном растворе.